ChaseDream
搜索
返回列表 发新帖
查看: 2026|回复: 5
打印 上一主题 下一主题

CD历史遗留问题,逻辑大虾请进 (之二)

[复制链接]
楼主
发表于 2003-9-7 10:42:00 | 只看该作者

CD历史遗留问题,逻辑大虾请进 (之二)


这几天,着重看了晴天猪的 Lsat CR 逻辑总结大贴,受益匪浅,如久旱逢甘霖,沐浴春风!建议所有朋友都先读一读该牛贴。现仍有若干未讨论彻底的题目或我仍糊涂的题目,请大虾们指点迷津!小虾米先拜过。

---------------------------------1-------------------------------------------
Q: LSAT SET1-4-22,23

22. It is illogical to infer a second and different effect from a cause which is known only by one particular effect. This is incorrect because the inferred effect must necessarily be produced by some different characteristic of the cause than is the observed effect, which already serves entirely to describe the cause.

Which one of the following arguments makes the same logical error as the one described by the author in the passage?
(A) An anonymous donor gave a thousand dollars to our historical society. I would guess that that individual also volunteers at the children’s hospital.
(B) The radioactive material caused a genetic mutation, which, in turn, caused the birth defect. Therefore, the radioactive material caused the birth defect.
(C) The tiny, unseen atom is the source of immense power. It must be its highly complex structure that produces this power.
(D) The city orchestra received more funds from the local government this year than ever before. Clearly this administration is more civic-minded than previous ones.
(E) If I heat water, which is a liquid, it evaporates. If I heat hundreds of other liquids like water, they evaporate. Therefore, if I heat any liquid like water, it will evaporate

答案是A,我选成了E,我认为逻辑错误是理由过于绝对,答案A如何解释?

我的问题是: 这道题目的文中意思是说 a =〉b, so a => c/d, 这样的逻辑是不对的,因为 c/d 可能除了a 以外还有别的原因; 如果是这样理解,答案就是 A.
还是说:a =>b, b=>c, so a=>c 是不对的,因为 c 有可能除了b 以外还有别的原因。
如果是这样理解,答案就是 B

----------------------------2---------------------------------------------
15. M: It is almost impossible to find a person between the ages of 85 an 90 who primarily uses the left hand.
Q: Seventy to ninety years ago, however, children were published for using their left hands to eat or to write and were forced to use their right hands.

Q's response serves to counter any use by M of the evidence about 85 to 90 year olds in supports of which one of the following hypotheses?

(A) Being born right-handed confers a survival advantage.
(B) Societal attitudes toward handedness differ at different times.
(C) Forcing a person to switch from a preferred hand is harmless.
(D) Handedness is a product of both genetic predisposition and social pressures.
(E) Physical habits learned in school often persist in old age.

答案A,我选D。

A: You misunderstood the question. It asks what M's argument is. D is the opinion and argument of Q's. A is M's argument

我的问题是:题目不是问的是Q's观点反对了M's 的观点来支持了以下那个假设吗?
in supports of which one of the following hypotheses?
即使是M's 的观点,我也楞没看出 A 就是答案。笨吧!


-----------------------------------3----------------------------------------

重问请教LSAT-4-I-7,9
请教LSAT-4-I-7,9
Two paleontologists, Dr Tyson and Dr. Rees, disagree over the interpretation of certain footprints that were left among other footprints in hardened volcanic ash at site G. Dr. Tyson claims they are clearly early hominid footprints since they show human characteristics: a squarish heel and a big toe immediately adjacent to the next toe. However, since the footprints indicate that if hominids made those prints they would have had to walk in an unexpected cross-stepping manner, by placing the left foot to the right of the right foot. Dr. Rees rejects Dr. Tyson's conclusion.

7. The disagreement between the two paleontologists is over which one of the following?

(A) the relative significance of various aspects of the evidence

(B) the assumption that early hominid footprints are distinguishable from other footprints

(C) the possibility of using the evidence of footprints to determine the gait of the creature that made those footprints

(D) the assumption that evidence from one paleontologic site is enough to support a conclusion

(E) the likelihood that early hominids would have walked upright on two feet

答案:A,我选了C。

最近做LSAT题,发现这种纯逻辑题反而做不好。本来不应是难题的。主要原因是经常有两个选项区别不清,做到最后,感觉是要再对应原文,再仔细区分两个选项。不知有没有什么好办法 ?

我的问题是:如果我不知道gait的意思--步态, 我肯定也会选C, 事实是我确实不知道,也确实做错了。有什么好办法能在不知道该单词的意思的情况下,仍然排除C?

沙发
发表于 2003-9-7 11:47:00 | 只看该作者
q: lsat set1-4-22,23

22. it is illogical to infer a second and different effect from a cause which is known only by one particular effect. this is incorrect because the inferred effect must necessarily be produced by some different characteristic of the cause than is the observed effect, which already serves entirely to describe the cause

我的问题是: 这道题目的文中意思是说 a =〉b, so a => c/d, 这样的逻辑是不对的,因为 c/d 可能除了a 以外还有别的原因; 如果是这样理解,答案就是 a.
还是说:a =>b, b=>c, so a=>c 是不对的,因为 c 有可能除了b 以外还有别的原因。
如果是这样理解,答案就是 b

我想你的第一种理解是对的啊。因为文中because the inferred effect must necessarily be produced by some different characteristic of the cause than is the observed effect, which already serves entirely to describe the cause. 不是第二种关系。
板凳
发表于 2003-9-7 11:53:00 | 只看该作者
15. m: it is almost impossible to find a person between the ages of 85 an 90 who primarily uses the left hand.
q: seventy to ninety years ago, however, children were published for using their left hands to eat or to write and were forced to use their right hands.

q's response serves to counter any use by m of the evidence about 85 to 90 year olds in supports of which one of the following hypotheses?

答案a,我选d。

a: you misunderstood the question. it asks what m's argument is. d is the opinion and argument of q's. a is m's argument

我的问题是:题目不是问的是q's观点反对了m's 的观点来支持了以下那个假设吗?
in supports of which one of the following hypotheses?
即使是m's 的观点,我也楞没看出 a 就是答案。笨吧!

b,c,d,e都明显不对,所以只好选a了。m的话中隐含着,这些人活到这么大,是因为用了右手。

(a) being born right-handed confers a survival advantage
地板
发表于 2003-9-7 11:59:00 | 只看该作者
我的问题是:如果我不知道gait的意思--步态, 我肯定也会选c, 事实是我确实不知道,也确实做错了。有什么好办法能在不知道该单词的意思的情况下,仍然排除c?

这就不好说,我想关健还是读懂题吧。两个人的分歧在于,一个认为只好有个什么样的脚印就可以证明存在。但另一个人认为光有这个证据不充分。所以a就是这个意思。
5#
 楼主| 发表于 2003-9-7 23:27:00 | 只看该作者

15. m: it is almost impossible to find a person between the ages of 85 an 90 who primarily uses the left hand.
q: seventy to ninety years ago, however, children were published for using their left hands to eat or to write and were forced to use their right hands.

q's response serves to counter any use by m of the evidence about 85 to 90 year olds in supports of which one of the following hypotheses?

答案a,我选d。

a: you misunderstood the question. it asks what m's argument is. d is the opinion and argument of q's. a is m's argument

我的问题是:题目不是问的是q's观点反对了m's 的观点来支持了以下那个假设吗?
in supports of which one of the following hypotheses?
即使是m's 的观点,我也楞没看出 a 就是答案。笨吧!

b,c,d,e都明显不对,所以只好选a了。m的话中隐含着,这些人活到这么大,是因为用了右手。

(a) being born right-handed confers a survival advantage

美眉,能不能帮我解释一下这个题目的问题的意思?q's response serves to counter any use by m of the evidence about 85 to 90 year olds in supports of which one of the following hypotheses? 在这里,in supports of which one of the following hypotheses 到底是support哪个? 谢谢了!

6#
 楼主| 发表于 2003-9-7 23:34:00 | 只看该作者

斑竹都到哪去了,看看嘛!!!
您需要登录后才可以回帖 登录 | 立即注册

Mark一下! 看一下! 顶楼主! 感谢分享! 快速回复:

手机版|ChaseDream|GMT+8, 2024-9-29 05:37
京公网安备11010202008513号 京ICP证101109号 京ICP备12012021号

ChaseDream 论坛

© 2003-2023 ChaseDream.com. All Rights Reserved.

返回顶部